Question

Consider a very (infinitesimally!) thin but massive rod, length L (total mass M), centered around the...

Consider a very (infinitesimally!) thin but massive rod, length L (total mass M), centered around the origin, sitting along the x-axis. (So the left end is at (-L/2, 0,0) and the right end is at (+L/2,0,0) Assume the mass density λ (which has units of kg/m)is not uniform, but instead varies linearly with distance from the origin, λ(x) = c|x|.

a) What is that constant “c” in terms of M and L? What is the direction of the gravitational field generated by this mass distribution at a point in space a distance z above the center of the rod, i.e. at (0,0,z) Explain your reasoning for the direction carefully.


b) Compute the gravitational field, ~g, at the point (0,0,z) by directly integrating Newton’s law of gravity, summing over all infinitesimal “chunks” of mass along the rod.


c) Compute the gravitational potential at the point (0,0,z) by directly integrating −Gdm/r, summing over all infinitesimal “chunks” dm along the rod. Then, take the z-component of the gradient of this potential to check that you agree with your result from the previous part.


d) In the limit of large z what do you expect for the functional form for gravitational potential? (Hint: Don’t just say it goes to zero! It’s a rod of mass M, when you’re far away what does it look like? How does it go to zero?) What does “large z” mean here? Use the binomial (or Taylor) expansion to verify that your formula does indeed give exactly what you expect. (Hint: you cannot Taylor expand in something BIG, you have to Taylor expand in something small.)
e) Can you use Gauss’ law to figure out the gravitational potential at the point (0,0,z)? (If so, do it and check your previous answers. If not, why not?)

0 0
Add a comment Improve this question Transcribed image text
Answer #1

eLoT us (Ganusi dan a thn but massive Rod of Mas M and Lonalh L placad as choun uin the Hare THE Linea h MASS DENSITY AND Tr2 7Y THE DIRECTION OF GRAUITATIONAL FIELD WIL BE TOwACD THE CENTRE OF THE Roo Auoma FIELD OVE To A SmALL ELEMENT OF THE RoD A0 -TAKING THE CONSTANTC Ouて。FINTEGen 2弦 2 go becom リ 2 O n +22 1리 NEGATIVE ー20,乙C て 12. THE FiELD Is ATRACTIUEC) THE GPAUITATIONAL POTENTIAL DuE TO IHE SMALL ELEMENT BECAUSE OF Poo WILLBE Twice OF GRAVITATIONAL POTENTIAL DUE TO ONE S HSame o リ2 7 1/ 2. 2. On SimPUFYING OR

Add a comment
Know the answer?
Add Answer to:
Consider a very (infinitesimally!) thin but massive rod, length L (total mass M), centered around the...
Your Answer:

Post as a guest

Your Name:

What's your source?

Earn Coins

Coins can be redeemed for fabulous gifts.

Not the answer you're looking for? Ask your own homework help question. Our experts will answer your question WITHIN MINUTES for Free.
Similar Homework Help Questions
  • Consider a thin uniform rod of mass M and length L, positioned along the z-axis with...

    Consider a thin uniform rod of mass M and length L, positioned along the z-axis with its ends at (0,0,0) and (0,0, - L). (3 marks) Calculate the force, F. it exerts on a mass m located at r = zk. (3 marks) Calculate the force, F, it exerts on another identical rod positioned with its ends at (0,0, L) and (0,0,2L). you should divide the second rod into small elements of length dz, to which the result from part...

  • The figure shows a thin rod, of length L = 2.10 m and negligible mass, that...

    The figure shows a thin rod, of length L = 2.10 m and negligible mass, that can pivot about one end to rotate in a vertical circle. A heavy ball of mass m = 9.20 kg is attached to the other end. The rod is pulled aside to angle θ0 = 22.0° and released with initial velocity = 0. As the ball descends to its lowest point, (a) how much work does the gravitational force do on it and (b)...

  • The figure shows a thin rod, of length L = 1.6 m and negligible mass, that...

    The figure shows a thin rod, of length L = 1.6 m and negligible mass, that can pivot about one end to rotate in a vertical circle. A heavy ball of mass m = 9.1 kg is attached to the other end. The rod is pulled aside to angle θ0 = 6.4° and released with initial velocity v Overscript right-arrow EndScripts Subscript 0 = 0. As the ball descends to its lowest point, (a) how much work does the gravitational...

  • A thin, uniform rod has length L and the linear density a (i.e. total mass M=al)....

    A thin, uniform rod has length L and the linear density a (i.e. total mass M=al). A point mass m is placed at distance x from one end of the rod, along the axis of the rod. Calculate the gravitational force of the rod on the point mass m. (Hint: element of the mass is dM = adx) -GmM/x? O-GmM/(L2-x2) -GmM/(x+.5L) -GmM/(x2+Lx)

  • A uniform thin rod of mass M and length L lies on the positive x-axis with...

    A uniform thin rod of mass M and length L lies on the positive x-axis with one end at the origin.Consider an element of the rod of length dx., and mass dm at point where 0<x<L. a) What is the gravitational field produced by the mass element of any value of X? b)Calculate the total gravitational field produced by the rod. C)Find the gravitational force on a point particle of mass m0 at x0. D) Show that for x0>>L the...

  • Consider a thin rod of length L and mass M situated with one end at the...

    Consider a thin rod of length L and mass M situated with one end at the origin x = 0 of the coordinate system as shown Mostly need help with part B. Thank you a) Four 1 kg boxes sit on a 5 m long uniform rod of mass 4 kg, such that the total mass of the system is 8 kg. The boxes are spaced 1 m apart, with the first box sitting at the left end of the...

  • 20. A thin rod of mass M and length L gravitationally interacts with a point mass...

    20. A thin rod of mass M and length L gravitationally interacts with a point mass m that is a perpendicular distance a away from its left end (see the figure). The rod is non-uniform, and its linear density (mass per unit length) increases with the distance from its left end according to 2(x) = 2Mx/L?, where x is the horizontal coordinate along the rod (so that x = 0 is at its left end and x=L is at its...

  • A very thin, straight, uniform rod has a length of 3.00 m and a total mass...

    A very thin, straight, uniform rod has a length of 3.00 m and a total mass of 7.00 kg. Treating the rod as essentially a line segment of mass (distributed uniformly), do the following: (i) Use integration to prove that the rod's center of mass is located at its center point. (Reminders: dmnds mass (and that axis is perpendicular to the rod). with the previous result-to calculate lemr the moment of inertia of the rod about an axis through one...

  • 1. A thin rod of length L and total mass M has a linear mass density...

    1. A thin rod of length L and total mass M has a linear mass density that varies with position as λ(x)-γ?, where x = 0 is located at the left end of the rod and γ has dimensions M/L3. ĮNote: requires calculus] (a) Find γ in terms of the total mass M and the length L. (b) Calculate the moment of inertia of this rod about an axis through its left end, oriented perpen dicular to the rod; expressed...

  • 0 A rod of length L and mass M is placed along the x-axis with one...

    0 A rod of length L and mass M is placed along the x-axis with one end at the origin, as shown in the figure above. The rod has linear mass density λ=en-x, where xis the distance from the origin. Which of the following gives the x-coordinate of the rod's center of mass? 2M 12 (B) I (C)江 4

ADVERTISEMENT
Free Homework Help App
Download From Google Play
Scan Your Homework
to Get Instant Free Answers
Need Online Homework Help?
Ask a Question
Get Answers For Free
Most questions answered within 3 hours.
ADVERTISEMENT
ADVERTISEMENT
ADVERTISEMENT